Wording Question
When going through the answer choices, I originally eliminated C because it used the word “many” ...
Jessica on September 29 at 04:20PM
  • June 2014 LSAT
  • SEC2
  • Q2
1
Reply
Answer choice A
P: the word "loophole" is a loaded partisan term that immediately implies wrong doing If "A" was...
Andrew on April 25 at 09:33PM
  • June 2014 LSAT
  • SEC2
  • Q6
4
Replies
Why is B wrong? There was a response to this in...
In the stimulus it sounds like the source of the lack of public confidence comes from the trivial...
Andrew on October 5, 2022
  • June 2014 LSAT
  • SEC2
  • Q10
2
Replies
"Some" as opposed to "most" in the context of t...
Hi, Answer-choice B would still be the correct choice, "most strengthen from the bunch," even ...
Mazen on July 16, 2022
  • June 2014 LSAT
  • SEC2
  • Q25
2
Replies
Answer-choice C
Hi, I anticipated B and ultimately selected it! C, however, did not give me trouble until I...
Mazen on July 16, 2022
  • June 2014 LSAT
  • SEC2
  • Q20
3
Replies
Whats wrong with D?
I was debating between B and D. I just can't even see why C is the correct answer.
on May 16, 2022
  • June 2014 LSAT
  • SEC2
  • Q2
1
Reply
Further explanation
Hi could you please explain why B? Thank you
Farnoush on September 22, 2021
  • June 2014 LSAT
  • SEC2
  • Q20
3
Replies
Explanation
Hi, can someone please explain why B is wrong?
Sophia on May 28, 2021
  • June 2014 LSAT
  • SEC2
  • Q10
5
Replies
Answer E
Can you please elaborate why E is correct? Thank you!
Brooke on January 8, 2021
  • June 2014 LSAT
  • SEC2
  • Q18
3
Replies
Why is D incorrect?
someone already asked this but I wanted to bump it up since no one has answered yet. Thanks!
Vennela on October 20, 2020
  • June 2014 LSAT
  • SEC2
  • Q12
2
Replies
Strengthen with Nec Premise
I am trying to better understand how to answer this question and negate the correct answer in ord...
Edgar on October 14, 2020
  • June 2014 LSAT
  • SEC2
  • Q12
4
Replies
Answer E
Hey why is answer choice E incorrect? Sorry I figured I should make myt own thread for my question.
Oscar on August 14, 2020
  • June 2014 LSAT
  • SEC2
  • Q16
2
Replies
Difference between A and C
Why is A wrong?
Ruchita on August 11, 2020
  • June 2014 LSAT
  • SEC2
  • Q14
1
Reply
Please explain C
I'm really confused as to y this is the answer.
Anonymous on July 26, 2020
  • June 2014 LSAT
  • SEC2
  • Q2
3
Replies
D wrong
Why is D wrong
Angelica on July 25, 2020
  • June 2014 LSAT
  • SEC2
  • Q5
1
Reply
Why B is wrong?
Why B is wrong?
mnv on July 19, 2020
  • June 2014 LSAT
  • SEC2
  • Q5
1
Reply
D vs E
would it be possible to get an explanation as to why E? i was stuck between both d and e. thank you!
on April 15, 2020
  • June 2014 LSAT
  • SEC2
  • Q7
1
Reply
Question
I get how C is right, but it doesn't automatically seem that way. Isn't the word choice rather st...
on April 4, 2020
  • June 2014 LSAT
  • SEC2
  • Q6
1
Reply
Answer
Please break down the answer and how it resolves the unknown
on April 2, 2020
  • June 2014 LSAT
  • SEC2
  • Q9
1
Reply
Answer
Why is c right
on April 2, 2020
  • June 2014 LSAT
  • SEC2
  • Q10
1
Reply